Prove the rotational invariance of the Laplace operator

Click For Summary
SUMMARY

The discussion focuses on proving the rotational invariance of the Laplace operator, represented mathematically as ##\Delta u=\partial_{x1}^2u+\partial_{x2}^2u+...+\partial_{xn}^2u##. The transformation under rotation is expressed through the matrix ##R##, where ##R^{-1}=R^T## is utilized to demonstrate the invariance. The conclusion is that the Laplacian remains unchanged under rotation, as shown by the equivalence ##\frac{\partial^2}{\partial \phi^2}=\frac{\partial^2}{\partial \phi'^2## when using cylindrical or spherical coordinates.

PREREQUISITES
  • Understanding of the Laplace operator in multiple dimensions
  • Familiarity with rotation matrices and their properties
  • Knowledge of cylindrical and spherical coordinate systems
  • Basic concepts of differential calculus and partial derivatives
NEXT STEPS
  • Study the properties of rotation matrices in linear algebra
  • Learn about the application of the Laplace operator in physics and engineering
  • Explore the derivation of the Laplace operator in cylindrical and spherical coordinates
  • Investigate the implications of rotational invariance in partial differential equations
USEFUL FOR

Mathematicians, physicists, and engineers interested in the properties of differential operators, particularly in the context of rotational symmetry and its applications in various fields.

docnet
Messages
796
Reaction score
486
Homework Statement
prove ##\Delta## is rotation invariant.
Relevant Equations
##\Delta##
Screen Shot 2021-01-24 at 10.08.21 PM.png

Hello, please lend me your wisdom.

##\Delta u=\partial_{x1}^2u+\partial_{x2}^2u+...+\partial_{xn}^2u##

##Rx=\left<r_{11}x_1+...r_{1n}x_n+...+r_{n1}x_1+...+r_{nn}x_n\right>##

##(\Delta u)(Rx)=(\partial_{x1}^2u+\partial_{x2}^2u+...+\partial_{xn}^2u)\left<r_{11}x_1+...r_{1n}x_n, ...,r_{n1}x_1+...+r_{nn}x_n\right>####u\circ R=##

##\begin{pmatrix}
(u)r_{11} & ... & (u)r_{1n} \\
... & ... & ... \\
(u)r_{n1} & ... & (u)r_{nn}
\end{pmatrix}##

##u\circ R x=\left<(u)r_{11}x_1+...(u)r_{1n}x_n+...+(u)r_{n1}x_1+...+(u)r_{nn}x_n\right>##

##\Delta u\circ R x= (\partial_{x1}^2+\partial_{x2}^2+...+\partial_{xn}^2)\left<(u)r_{11}x_1+...(u)r_{1n}x_n+...+(u)r_{n1}x_1+...+(u)r_{nn}x_n\right>=##

##(\partial_{x1}^2u+\partial_{x2}^2u+...+\partial_{xn}^2u)\left<r_{11}x_1+...r_{1n}x_n, ...,r_{n1}x_1+...+r_{nn}x_n\right>=(\Delta u)(Rx)##

I think ##u\circ R## does not mean ##(u) (R)## and I just showed the same calculation twice. How do I use the information ##R^{-1}=R^T## to prove this case?
 
Physics news on Phys.org
Let us take z-axis as axis of rotation of angle ##\alpha## and take cylindrical coordinates or spherical coordinates. For the rotation
\phi&#039;=\phi + \alpha
, we get rotation invariance of Laplacian because
\frac{\partial^2}{\partial \phi^2}=\frac{\partial^2}{\partial \phi&#039;^2}
 
  • Like
Likes docnet and dRic2
Question: A clock's minute hand has length 4 and its hour hand has length 3. What is the distance between the tips at the moment when it is increasing most rapidly?(Putnam Exam Question) Answer: Making assumption that both the hands moves at constant angular velocities, the answer is ## \sqrt{7} .## But don't you think this assumption is somewhat doubtful and wrong?

Similar threads

  • · Replies 1 ·
Replies
1
Views
3K
  • · Replies 30 ·
2
Replies
30
Views
2K
  • · Replies 16 ·
Replies
16
Views
1K
  • · Replies 8 ·
Replies
8
Views
3K
Replies
27
Views
2K
Replies
14
Views
2K
  • · Replies 1 ·
Replies
1
Views
2K
  • · Replies 15 ·
Replies
15
Views
2K
Replies
2
Views
2K
  • · Replies 8 ·
Replies
8
Views
4K